Adult Gerontology - Endocrine Q&A, Endocrine

Pataasin ang iyong marka sa homework at exams ngayon gamit ang Quizwiz!

Question 20. Eunice, age 32, has type 2 diabetes. She said she heard she should take an aspirin a day after she reaches menopause for its cardioprotective action. She does not have coronary artery disease, but her father does. How do you respond? 1. "You're right. Your hormones protect you against coronary artery disease until menopause; then you should start on aspirin therapy." 2. "The American Diabetes Association recommends that you start on low-dose aspirin therapy now." 3. "Aspirin therapy is recommended for all patients over age 55 as a precautionary measure." 4. "If you maintain good glycemic control, you don't need aspirin therapy."

2. "The American Diabetes Association recommends that you start on low-dose aspirin therapy now." Option 1: The American Diabetes Association (ADA) recommends aspirin therapy as a primary prevention strategy in high-risk men and women with diabetes who have a family history of coronary heart disease. Option 2: The American Diabetes Association's position statement on aspirin therapy in patients with diabetes recommends low-dose (81 mg) aspirin use as a secondary prevention strategy in men and women with diabetes who have evidence of large-vessel disease, such as a history of myocardial infarction, vascular bypass procedures, and stroke, and have no contraindications for the use of aspirin. They also recommend aspirin therapy as a primary prevention strategy in high-risk men and women with type 1 or type 2 diabetes who have a family history of coronary heart disease and for individuals who smoke, are hypertensive or obese, or who have albuminuria, cholesterol levels greater than 200 mg/dL, low-density lipoprotein cholesterol levels greater than 130 mg/dL, high-density lipoprotein cholesterol levels less than 40 mg/dL, and triglyceride levels greater than 250 mg/dL. Option 3: Aspirin therapy is not recommended as a precautionary measure in all patients over age 55. Option 4: The American Diabetes Association (ADA) recommends aspirin therapy as a primary prevention strategy in high-risk men and women with diabetes who have a family history of coronary heart disease.

Question 1. Martin, age 62, has acute nontransient abdominal pain that grows steadily worse in the epigastric area and radiates straight through to the back. The pain has lasted for days. He is also complaining of nausea, vomiting, sweating, weakness, and pallor. Physical examination reveals abdominal tenderness and distention and a low-grade fever. What do you suspect? 1. Cholecystitis. 2. Acute pancreatitis. 3. Cirrhosis. 4. Cushing syndrome.

2. Acute pancreatitis. Option 1: The pain with cholecystitis is in the upper right quadrant and is intermittent, usually after a fatty meal. Option 2: Acute pancreatitis is an inflammation of the pancreas caused by the release of activated pancreatic enzymes into the surrounding parenchyma, with subsequent destruction of tissue, blood vessels, and supporting structures. Although pancreatitis may be acute or chronic, acute symptoms include continuous abdominal pain of several days' duration that increases in the epigastric area and radiates to the back, nausea, vomiting, sweating, weakness, pallor, abdominal tenderness, distention, and low-grade fever. Pancreatitis occurs primarily in middle-aged adults and slightly more often in women than in men. Option 3: The gastrointestinal (GI) manifestations of cirrhosis include parotid enlargement, esophageal or rectal varices, peptic ulcers, and gastritis. Option 4: The clinical manifestation of Cushing syndrome related to the gastrointestinal (GI) system is a peptic ulcer, which would result in intermittent pain related to meals.

Which of the following body mass index (BMI) values defines class 1 obesity? A. 30 B. 25 C. 40 D. 35

A. 30 Option 1: Class 1 obesity is defined as a BMI of 30 to less than 35. Option 2: Overweight is defined as a BMI of 25 to less than 30. Option 3: Class 3 obesity is defined as a BMI of 40 or higher. Option 4: Class 2 obesity is defined as a BMI of 35 to less than 40.

Question 1. Marsha, age 24, is preparing for radioactive iodine therapy for her Graves disease. Which test must she undergo first? 1. Beta-human chorionic gonadotropin. 2. Basal metabolism rate. 3. Lithium level. 4. Serum calcium.

1. Beta-human chorionic gonadotropin. Option 1 Radioactive iodine therapy is the most commonly used treatment in the United States for Graves disease (hyperthyroidism); however, it is contraindicated during pregnancy. Therefore, for women, a pregnancy test (beta-human chorionic gonadotropin) needs to be performed before initiating therapy. Women of childbearing age should also be told to delay conception for a few months after radioactive iodine therapy. It is also contraindicated in women who are breastfeeding. Older adults or clients at risk of developing cardiac complications may be pretreated with antithyroid drugs (ATDs) before therapy to deplete the thyroid gland of stored hormone, thereby minimizing the risk of exacerbation of hyperthyroidism because of radioactive iodine (131I)-induced thyroiditis. Option 2: Marsha's basal metabolism rate will be affected by her Graves disease but has no bearing on her preparation for radioactive iodine therapy. Option 3: Lithium levels are usually not performed before radioactive iodine therapy. They may be done if lithium is being used to block the release of thyroid hormone from the thyroid gland in clients who are intolerant of antithyroid drugs (ATDs). Option 4: Although parathyroid hormone secretion is dependent on the serum calcium level, it is usually not necessary to obtain a serum calcium level measurement before radioactive iodine therapy.

Question 2. The most common cause of hyperthyroidism is: 1. Graves disease. 2. A toxic uninodular goiter. 3. Subacute thyroiditis. 4. A pituitary tumor.

1. Graves disease Option 1: The most common cause of hyperthyroidism is an autoimmune condition known as Graves disease, which accounts for 90% of hyperthyroid conditions in young adults. Option 2: A toxic uninodular goiter is the second most common cause of hyperthyroidism. Option 3: Subacute thyroiditis is a less common cause of hyperthyroidism. Option 4: A pituitary tumor is a less common cause of hyperthyroidism.

What is the most common cause of Cushing disease? 1. Pituitary adenoma 2. Prednisone use 3. Adrenal tumor 4. Nonpituitary tumor

1. Pituitary adenoma option 1: Pituitary adenomas are the cause of 70% of cases of Cushing disease; they secrete excessive adrenocorticotropic hormone (ACTH). Option 2: Exogenous glucocorticoids are the second most common cause of Cushing disease. Option 3: Adrenal tumors can cause Cushing disease but are responsible for only 30% of adrenocorticotropic hormone (ACTH)-independent cases. ACTH-independent Cushing disease is less common than ACTH-dependent Cushing disease. Option 4: Nonpituitary tumors that secrete adrenocorticotropic hormone (ACTH) are rare.

Question 2. What is the primary pathological irregularity associated with diabetes mellitus type 1? 1. Nonfunctioning beta cells. 2. Insulin resistance. 3. Overproducing beta cells. 4. Elevated free fatty acids in the plasma.

1. Nonfunctioning beta cells. Option 1: Beta cells in the pancreas secrete insulin. When they don't work, your body can't lower its blood sugar; this is the pathophysiology of diabetes mellitus type 1. Option 2: This is most commonly associated with diabetes mellitus type 2. Option 3: This is a beta cell reaction associated with hyperglycemia. Option 4: This is a product of hyperglycemia, not the cause of diabetes mellitus type 1.

Question 19. Lynne has Cushing syndrome. You would expect her to have or develop: 1. Onychomycosis. 2. Generalized increased pigmentation of the skin. 3. Hair loss. 4. Excitability and nervousness.

1. Onychomycosis. Option 1: Cushing syndrome results in an excessive amount of adrenocorticotropic hormone, which stimulates the secretion of glucocorticoids, mineralocorticoids, and androgenic steroids from the adrenal cortex. In the presence of excessive cortisol, fungal infections of the skin, nails, and oral mucosa, such as onychomycosis and tinea versicolor, are common and skin wounds heal very slowly. Option 2: Addison disease, which is a deficiency in the secretion of adrenocortical hormones, usually results in increased pigmentation of the skin in its entirety. Option 3: Other symptoms of Cushing syndrome include excessive hair growth (not hair loss). Option 4: Other symptoms of Cushing syndrome include fatigue and weakness (not excitability and nervousness).

Question 4. Betty, age 40, has had type 1 diabetes for 20 years and takes a combination of neutral protamine Hagedorn (NPH) and regular insulin every day. She comes to the office because she has developed a severe upper respiratory infection with chills, fever, and production of yellow sputum. Because of her acute infection, you know that Betty is likely to require: 1. A decrease in her daily insulin dosage. 2. An increase in her daily insulin dosage. 3. A high-calorie dietary intake and no insulin change. 4. A change in her insulin from NPH to insulin aspart (NovoLog).

2. An increase in her daily insulin dosage. Option 1: Betty is likely to require an increase in her daily insulin dosage. Option 2: For clients with diabetes requiring insulin, an increase in their daily insulin dosage is usually required in the presence of an acute infection. Betty should begin by increasing her regular insulin dose by just 2 units and then monitoring her blood sugar level. Option 3: Betty is likely to require an increase in her daily insulin dosage and no increase in dietary caloric intake. Option 4: At some point, a more physiologic insulin regimen (such as basal-bolus) might be considered.

Question 5. A patient presents to your primary care office with abnormal lab results. On physical exam, you tap the patient's facial nerve around the zygomatic arch, just anterior to the earlobe. This describes which of the following tests and is associated with which of the following lab abnormalities? 1. Trousseau sign, hypocalcemia. 2. Chvostek sign, hypocalcemia. 3. Chvostek sign, hypercalcemia. 4. Lachman test, hypercalcemia.

2. Chvostek sign, hypocalcemia. Option 1: Trousseau sign is positive in hypocalcemia but is described as spasms of the hand and wrist following inflation of a blood pressure cuff in the arm. Option 2: The test described in the question is positive in someone with hypocalcemia and is called Chvostek sign. Option 3: The test described in the question is consistent with Chvostek sign but is not consistent with hypercalcemia; it is consistent with hypocalcemia. Option 4: A Lachman test is positive in the presence of anterior cruciate ligament tears in the knee.

Question 10. Mr. Reynolds is on the antithyroid drug (ATD) methimazole (Tapazole), so you make it a point to check his: 1. Glycated hemoglobin (HbA1c). 2. Complete blood count (CBC) and liver transaminases. 3. Uric acid level. 4. Total thyroxine (T4).

2. Complete blood count (CBC) and liver transaminases. Option 1: HbA1c is monitored in clients with diabetes. Option 2: ATDs can cause agranulocytosis and hepatic injury; therefore, CBC and liver studies should be done. Option 3: Uric acid is monitored in clients with gout. Option 4: Thyroid function tests are needed in patients taking ATDs; however, thyroid-stimulating hormone (TSH) and free T4 are preferred.

Question 7. Juanita, age 23, complains of palpitations that started a few weeks ago; they occur 2 to 4 times a day and last 5 to 10 minutes. She feels nervous and is having trouble sleeping. Her stools have been frequent (1-3 per day) and loose. She is taking levothyroxine 150 µg daily. Her labs indicate free thyroxine (T4) 2.28 and thyroid-stimulating hormone (TSH) 0.022. She has a history of Graves disease and had radioactive iodine (RAI) treatment a few months ago. She has been on thyroid replacement for 2 months. Based on these data, you decide to: 1. Increase the levothyroxine dosage. 2. Decrease the levothyroxine dosage. 3. Keep the dosage the same. 4. Start propranolol every 8 hours.

2. Decrease the levothyroxine dosage. Option 1: A lower dosage of levothyroxine will probably correct Juanita's symptoms. Option 2: It appears that she may be overcorrected. The usual dosage of thyroid replacement is 1.6 µg/kg/d. She could skip a dose and then resume at a lower dosage of 125 to 137 µg per day. In an older individual, the lower dose would be preferred because overcorrection can lead to atrial fibrillation. She should take the levothyroxine on an empty stomach with a full glass of water and wait 30 minutes before eating for maximum absorption. Option 3: A lower dosage of levothyroxine will probably correct Juanita's symptoms. Option 4: Propranolol, which may help palpitations in some patients, is not necessary here, as the correct dosage of levothyroxine will probably correct Juanita's symptoms.

Question 6. Harriet, age 62, has type 1 diabetes that is well controlled by insulin. Recently, she has been having marital difficulties that have left her emotionally upset. As a result of this stress, it is possible that she will: 1. Have an insulin reaction more readily than usual. 2. Have an increased blood sugar level. 3. Need less daily insulin. 4. Need more carbohydrates.

2. Have an increased blood sugar level. Option 1: Harriet will not have an insulin reaction (such as hypoglycemia) more readily than usual. Option 2: Stress causes the adrenal glands to secrete more cortisol, which leads to gluconeogenesis and insulin antagonism, raising the blood sugar. It is possible, then, that Harriet will have an increased blood sugar level. She will not need less daily insulin or more carbohydrates and will not have an insulin reaction (such as hypoglycemia) more readily than usual. Harriet may, in fact, need to increase her insulin use. Option 3: She will not need less daily insulin; she may need to increase her insulin use. Option 4: She will not need more carbohydrates; it is possible she will have an increased blood sugar level.

Question 1. A patient presents to your primary care office complaining of polydipsia, polyuria, and polyphagia. Which of the following diagnoses would not be in your differential diagnosis? 1. Diabetes mellitus (DM). 2. Diabetes insipidus (DI). 3. Psychiatric disorders. 4. Hyperthyroidism.

4. Hyperthyroidism. Option 1: Polydipsia, polyuria, and polyphagia are common presenting complaints in DM. Option 2: Polydipsia, polyuria, and polyphagia are common presenting complaints in DI. Option 3: Many psychiatric disorders can have polydipsia, polyuria, and polyphagia as presenting symptoms. Option 4: Symptoms of hyperthyroidism could include polyphagia due to increased metabolism, but symptoms generally are weight loss, tachycardia, and other physical manifestations of increased metabolism.

Question 3. Mark has type 1 diabetes and has mild hyperglycemia. What effect does physical activity (exercise) have on his blood glucose level? 1. It may cause it to vary a little. 2. It may decrease it. 3. It may elevate it. 4. It may fluctuate greatly either way.

2. It may decrease it. Option 1: For individuals without diabetes, the blood glucose level generally varies little during physical activity unless the activity is intense and of very long duration, such as marathon running. Option 2: Clients with insulin-dependent diabetes mellitus (IDDM)—ie, type 1 diabetes—who have mild hyperglycemia may experience a drop in their blood glucose level during physical activity, whereas those with marked hyperglycemia may experience a rise in their blood glucose level. Clients with IDDM should check their blood glucose level before exercising and refrain from exercising if their level is too high (greater than 300 mg/dL). Option 3: Clients with marked hyperglycemia may experience a rise in their blood glucose level during physical activity. Option 4: Since Mark has mild hyperglycemia, his blood glucose level may drop during physical activity.

Question 17. A client with hyperthyroidism presents with a complaint of a "gritty" feeling in her eyes. Over the past week, her visual acuity has diminished, and her ability to see colors has changed. She also has a feeling of pressure behind her eyes. The next step for the nurse practitioner is to: 1.Order a thyroid ultrasound. 2.Refer the client for immediate evaluation by an ophthalmologist. 3.Order a total thyroxine (T4). 4.Prescribe a beta-adrenergic blocker.

2.Refer the client for immediate evaluation by an ophthalmologist. The practitioner should refer the client for an immediate evaluation by an ophthalmologist. Clinically recognized Graves ophthalmopathy occurs in about 50% of cases of Graves disease. A client with Graves orbitopathy with these complaints is at risk of blindness if there is compression of the optic nerve. Additional symptoms include photophobia and diplopia. Autoantibodies present in Graves disease can cause increased muscle thickness in the eye, leading to edema and compression of the optic nerve. Fundal exam may reveal disk swelling. This is an emergency situation that may require hospitalization and treatment with prednisone to diminish the inflammation. Artificial tears are also helpful. In 75% of clients, the onset of Graves orbitopathy occurs within a year before or after the diagnosis of thyrotoxicosis but can sometimes precede or follow thyrotoxicosis by several years.

Morris has had type 1 diabetes for 10 years. Several recent urinalysis reports have shown microalbuminuria. Your next step would be to: 1.Order a 24-hour urinalysis. 2.Start him on an angiotensin-converting enzyme (ACE) inhibitor. 3.Stress the importance of strict blood sugar control. 4.Send him to a dietitian because he obviously has not been following his diet.

2.Start him on an angiotensin-converting enzyme (ACE) inhibitor. Morris should be started on an ACE inhibitor such as enalapril (Vasotec). ACE inhibitors offer renoprotective effects by reducing intraglomerular pressure. They do this by inhibiting the renin-angiotensin system, which causes efferent dilation, and by improving glomerular permeability, which causes a reduction of glomerulosclerosis. ACE inhibitors also have this beneficial effect on clients with diabetes who are normotensive and even hypotensive. Diabetic nephropathy is the leading cause of end-stage renal disease in the United States. Monitoring for microalbuminuria is one method for identifying early nephropathy.

Question 12. Your client with diabetes asks you about insulin glargine (Lantus). You tell her that: 1. It may be administered subcutaneously at home or intravenously in the hospital if need be. 2. The onset of action is 15 minutes. 3. Insulin glargine (Lantus) stays in your system for 24 hours. 4. It can be mixed with any other insulin.

3. Insulin glargine (Lantus) stays in your system for 24 hours Option 1: Insulin glargine (Lantus) must be administered subcutaneously, not intravenously. Regular insulin may be administered by the intravenous route. The newer insulin analogues, such as insulin aspart (NovoLog), are also approved for intravenous use. Option 2: Insulin glargine (Lantus) has an onset of action of just over 1 hour. Option 3: Insulin glargine (Lantus) has an onset of action of just over 1 hour and stays in the system for 24 hours. Option 4: Insulin glargine (Lantus) and insulin detemir (Levemir) may not be mixed with any other insulin.

Question 17. Which of the following would not confirm a diagnosis of diabetes? 1. Glycated hemoglobin (HbA1c) of 7.0. 2. Fasting glucose of 155. 3. Random glucose of 198. 4. Oral glucose tolerance test with a plasma glucose of 250.

3. Random glucose of 198. Option 1: An HbA1c greater than 6.5 is diagnostic of diabetes. Option 2: A fasting glucose greater than 126 is diagnostic of diabetes. Option 3: A random glucose greater than 200, with associated symptoms such as weight loss, polyuria, and polydipsia, is diagnostic of diabetes. Option 4: An oral glucose tolerance test with a plasma glucose greater than 200 is diagnostic of diabetes.

Which of the following would not confirm a diagnosis of diabetes? 1. Glycated hemoglobin (HbA1c) of 7.0. 2. Fasting glucose of 155. 3. Random glucose of 198. 4. Oral glucose tolerance test with a plasma glucose of 250.

3. Random glucose of 198. Option 1:An HbA1c greater than 6.5 is diagnostic of diabetes.Option 2:A fasting glucose greater than 126 is diagnostic of diabetes.Option 3:A random glucose greater than 200, with associated symptoms such as weight loss, polyuria, and polydipsia, is diagnostic of diabetes.Option 4:An oral glucose tolerance test with a plasma glucose greater than 200 is diagnostic of diabetes.

Question 5. Tamika, who has diabetes, states that she heard fiber is especially good to include in her diet. How do you respond? 1."Fiber is important in all diets." 2."Too much fiber interferes with insulin, so include only a moderate amount in your diet." 3."Fiber, especially soluble fiber, helps improve carbohydrate metabolism, so it is more important in the diet of persons with diabetes." 4."You get just the amount of fiber you need with a normal diet."

3."Fiber, especially soluble fiber, helps improve carbohydrate metabolism, so it is more important in the diet of persons with diabetes." Option 1: While fiber is important in all diets, this answer does not address why fiber is especially good to include in the diet of a patient with diabetes. Option 2: A diet high in fiber, especially soluble fiber, helps improve carbohydrate metabolism. Option 3: Fiber is important in the dietary management of diabetes. A diet high in fiber, especially soluble fiber, helps improve carbohydrate metabolism and lowers both total cholesterol and low-density lipoprotein cholesterol. Soluble fiber is found in dried beans, oats, and barley as well as some vegetables and fruits (peas, corn, zucchini, cauliflower, broccoli, prunes, pears, apples, bananas, and oranges). Option 4: It should not be assumed that individuals get enough fiber in their diet because most dietary habits are not perfect. An intake of 20 to 30 g of fiber per day is recommended.

Question 4. Joy has gout. In teaching her about her disease, which food do you tell her is allowed in the diet? 1.Asparagus. 2.Beans. 3.Broccoli. 4.Mushrooms.

3.Broccoli. Option 1: Asparagus is high in purine. Option 2: Beans are high in purine. Option 3: Foods high in purine should be avoided by clients with gout. Broccoli is not high in purine. Foods high in purine include all meats and seafood, meat extracts and gravies, yeast and yeast extracts, beans, peas, lentils, oatmeal, spinach, asparagus, cauliflower, and mushrooms. Wine and alcohol in excessive amounts impair the ability of the kidneys to excrete uric acid and should be used in moderation. Option 4: Mushrooms are high in purine.

Question 3. Leah, age 70, has had diabetes for many years. When teaching her about foot care, you want to stress: 1.That her calluses will protect her from infection. 2.The need to assess the bottom of her feet carefully after walking barefoot. 3.That painless ulcerations might occur and feet should be examined with a mirror. 4.That mild pain is to be expected because of neuropathy.

3.That painless ulcerations might occur and feet should be examined with a mirror. Option 1: She should try to avoid the development of calluses because preparations used to remove them are very caustic. Option 2: Leah should not be walking barefoot because her sensation is probably decreased as a result of neuropathy. Option 3: Painless ulcerations are very common in clients with diabetes, and the only way to assess for them in the feet is for clients to use a mirror to examine the bottoms of their feet. Option 4: Sensation may be decreased as a result of neuropathy.

Dan, age 45, is obese and has type 2 diabetes. He has been having trouble getting his glycohemoglobin under control. He has heard that exenatide (Byetta) causes weight loss and wants to try it. What do you tell him? 1. "Let's adjust your oral antidiabetic agents instead." 2. "That's a myth. People usually change their eating habits when taking this, and that's what causes the weight loss." 3. "With type 2 diabetes, you never want to be on injectable insulin." 4. "Let's try it. Your glycohemoglobin will be lowered and you may lose weight."

4. "Let's try it. Your glycohemoglobin will be lowered and you may lose weight." Option 1: Adjusting the client's oral antidiabetic agents may not be as effective, but these drugs are less expensive. Option 2: Exenatide (Byetta) can cause weight loss in some individuals. Option 3: Glucagon-like peptide-1 (GLP-1) injectables like exenatide are not injectable insulins. Option 4: Unlike many oral antidiabetic agents, injectable exenatide (Byetta) can cause weight loss in some individuals. The active ingredient is a protein that encourages digestion and the production of insulin. Glucagon-like peptide-1 (GLP-1) injectables like exenatide and liraglutide (Victoza) may be used as an adjunct to diet and exercise to improve glycemic control in adults with type 2 diabetes mellitus.

Question 3. When you inspect the integumentary system of clients with endocrine disorders, a finding of coarse hair may be an indicator of: 1. Addison disease. 2. Diabetes mellitus. 3. Cushing syndrome. 4. Hypothyroidism.

4. Hypothyroidism.

Question 14. Jeremiah, age 72, has gout and is obese. When teaching him about diet, which of the following do you tell him? 1. "Beer and wine are okay because they have no effect on uric acid." 2. "Keeping your weight stable, even if you are a little overweight, is better than fluctuating." 3. "You must go on a restricted, very low calorie diet to effect immediate change." 4. "Fluid intake should exceed three thousand milliliters daily to prevent formation of uric acid kidney stones."

4. "Fluid intake should exceed three thousand milliliters daily to prevent formation of uric acid kidney stones." Option 1: Because both wine and beer in excessive amounts impair the ability of the kidneys to excrete uric acid, they should be used in moderation. Clients must be aware that binge drinking may provoke an acute attack. Option 2: If the client is obese, weight loss should be encouraged because loss of excess body fat may normalize serum uric acid without pharmacological intervention. Weight loss will also decrease stress on weight-bearing joints. Option 3: Caution regarding severe, rapid weight loss should be given because secondary hyperuricemia may result. A restricted, very low calorie diet may precipitate an acute attack. Option 4: Fluid intake should exceed 3000 mL daily to prevent formation of uric acid kidney stones. Clients should avoid dehydration because it may precipitate an acute attack.

Sadie, age 40, has just been given a diagnosis of Graves disease. She has recently lost 25 lb, has palpitations, is very irritable, feels very warm, and has a noticeable bulge on her neck. The most likely cause of her increased thyroid function is: A. Hyperplasia of the thyroid B. An anterior pituitary tumor C. A thyroid carcinoma D. An autoimmune response

4. An autoimmune response Option 1: Hyperplasia of the thyroid results from hyperthyroidism; it does not cause it. Option 2: Although pituitary tumors can cause hyperthyroidism, they do not cause Graves disease. Option 3: Although thyroid carcinoma can cause hyperthyroidism, it does not cause Graves disease. Option 4: Graves disease is the result of an autoimmune response wherein antibodies are produced that act against the body's own organs and tissues. Thyroid-stimulating immunoglobulins are found in 95% of people with Graves disease and are evidence of this autoimmune process.

Question 13. Sara, age 40, has diabetes and is now experiencing anhidrosis on the hands and feet, increased sweating on the face and trunk, dysphagia, anorexia, and heartburn. Which complication of diabetes do you suspect? 1. Macrocirculation changes. 2. Microcirculation changes. 3. Peripheral neuropathies. 4. Autonomic neuropathies.

4. Autonomic neuropathies. Option 1: Macrocirculation changes include an early onset of atherosclerosis and peripheral vascular insufficiency with claudication, ulcerations, and gangrene of the legs. Option 2: Microcirculation changes include diabetic retinopathy with retinal ischemia and loss of vision and diabetic nephropathy with hypertension, albuminuria, edema, and progressive renal failure. Option 3: Peripheral neuropathies include changes in sensation in the feet and hands; palsy of cranial nerve III with headache, eye pain, and inability to move the eye up, down, or to the middle; pain or loss of cutaneous sensation over the chest; and motor and sensory deficits in the anterior thigh and medial calf. Option 4: Autonomic neuropathies include anhidrosis (absence of sweating) on the hands and feet, increased sweating on the face and trunk, dysphagia, anorexia, heartburn, constricted pupils, nausea and vomiting, constipation, and diabetic diarrhea.

Question 8. Which of the following would not be ordered on a regular basis to evaluate diabetic patients for end organ damage associated with diabetes? 1. Ophthalmology evaluation. 2. Lipid panel. 3. Urinalysis and basic metabolic panel (BMP). 4. Complete blood count (CBC).

4. Complete blood count (CBC). Option 1: The general recommendation for diabetic patients is to evaluate for diabetic retinopathy at diagnosis and at least every 2 years thereafter. Option 2: Hyperlipidemia is common in diabetes, and a lipid panel should be ordered annually. Option 3: A urinalysis and BMP should be ordered regularly for the evaluation of kidney damage associated with diabetes. Option 4: A CBC should be done in all patients at an annual physical, but diabetics don't typically have end organ damage that would be evident on a CBC.

Question 9. A client with diabetes on a sulfonylurea and metformin with a glycated hemoglobin (HbA1c) of 6.5% is complaining of episodes of low blood sugar. Which of the following changes would be the most appropriate? 1. Decreasing the dosage of the metformin. 2. Discontinuing the metformin. 3. Increasing carbohydrate intake. 4. Decreasing the dosage of the sulfonylurea.

4. Decreasing the dosage of the sulfonylurea. Option 1: There is less risk of hypoglycemia with metformin than with sulfonylureas. Option 2: There is less risk of hypoglycemia with metformin than with sulfonylureas. Option 3: Increasing the carbohydrate intake is never a good choice in a patient with diabetes. Option 4: Metformin, dipeptidyl peptidase-4 (DPP4) inhibitors such as sitagliptin (Januvia), and incretin mimetics such as exenatide (Byetta) are gaining favor over sulfonylureas because the risk of hypoglycemia is less than with sulfonylureas.

Which of the following statements about diabetes mellitus is untrue? 1. Diabetes mellitus is the most common etiology of renal failure in the United States. 2. Diabetes mellitus is the most common endocrine disorder in the United States. 3. Diabetes mellitus is characterized by impaired insulin secretion and insulin action. 4. Diabetes mellitus is curable.

4. Diabetes mellitus is curable.

Question 18. After an oral cholecystogram, Sam complains of burning on urination. This is because of: 1. A mild reaction to the contrast medium. 2. Biliary obstruction. 3. Contraction of the gallbladder. 4. The presence of dye in the urine.

4. The presence of dye in the urine. Option 1: A reaction to the contrast medium would produce symptoms such as urticaria, nausea, vomiting, and dyspnea. Option 2: An oral cholecystogram is done to assess for biliary obstruction. Option 3: To obtain a better reading during a cholecystogram, contraction of the gallbladder may be desirable and may be accomplished by having the client consume a high-fat meal during the procedure. Option 4: After an oral cholecystogram, some people experience burning on urination because of the presence of dye in the urine. This is helped by forcing fluids.

Question 2. Ben, a client with type 1 diabetes, is hospitalized with an admitting diagnosis of diabetic ketoacidosis (DKA). Which of the following signs and symptoms would be consistent with this condition? 1.Hypoglycemia and glycosuria. 2.Decreased respiratory rate with shallow respirations. 3.Polydipsia and an increased blood pH. 4.Ketonuria and polyuria.

4.Ketonuria and polyuria. Option 1: Hyperglycemia, not hypoglycemia, is a symptom of DKA. Option 2: Very deep, not shallow, respirations are a symptom of DKA. Option 3: A decreased, not increased, blood pH is a symptom of DKA. Option 4: Signs and symptoms of diabetic ketoacidosis include Kussmaul breathing (very deep respiratory movements), hyperglycemia, glycosuria, polyuria, polydipsia, anorexia, and headache, as well as ketonuria and a decreased blood pH.

A client presents with clinical manifestations of hyperthyroidism. The differential diagnosis includes Graves disease and subacute thyroiditis. Which of the following findings is consistent with subacute thyroiditis? 1. A 24-hour radioactive iodine uptake (RAIU) demonstrating increased uptake. 2. A 24-hour radioactive iodine uptake (RAIU) demonstrating decreased uptake. 3. A 24-hour radioactive iodine uptake (RAIU) demonstrating a "cold" nodule. 4. A fine-needle aspiration is required for diagnosis of subacute thyroiditis.

A 24-hour radioactive iodine uptake (RAIU) demonstrating increased uptake A 24-hour radioactive iodine uptake helps establish the functional status of the thyroid gland. A patient with Grave's disease will demonstrate increased uptake of the radioactive iodine. In subacute thyroiditis, the radioactive iodine uptake is low. Subacute thyroiditis produces symptoms of thyrotoxicosis via the release of preformed thyroid hormones in response to an inflammatory response after an acute infection. Subacute thyroiditis is usually self-limiting and is treated with NSAIDs and beta-adrenergic antagonists and in severe cases corticosteroids. Thyroid function should be monitored because a state of hypothyroidism can occur. The Discovery of a "cold" nodule on RAIU is suspicious for malignancy and requires a fine-needle aspiration and biopsy.

You suspect that Sharon has hypoparathyroidism because, in addition to her other signs and symptoms, she has: A. Elevated serum phosphate levels B. Elevated serum calcium levels C. Decreased neuromuscular activity D. Increased bone resorption, as implied by her bone density test

A. Elevated serum phosphate levels Signs of hypoparathyroidism include elevated serum phosphate levels; decreased serum calcium levels; increased neuromuscular activity, which may progress to tetany; decreased bone resorption; hypocalciuria; and hypophosphatemia.

A 55-year-old Asian male presents with a history of severe left great toe pain. He states he cannot even touch the toe with a sheet without it causing pain. He denies trauma but states he cannot ambulate without pain. He admits to drinking alcohol but not to excess. On physical exam, he has normal vital signs, and you note erythema of the great toe at the interphalangeal (IP) joint. Which of the following is the gold standard for diagnosis of this problem? A. Joint aspiration with crystal analysis B. Serum uric acid level C. X-ray D. Diagnosis is made with physical exam only

A. Joint aspiration with crystal analysis

A low thyroid-stimulating hormone (TSH) can lead to: A. Osteoporosis B. Weight gain C. Bradycardia D. Brittle hair

A. Osteoporosis Hyperthyroidism presents with a suppressed TSH and elevated free thyroxine (T4). Manifestations include weight loss, tachycardia, diarrhea, anxiety, and warm, silky skin. The increased metabolic state of hyperthyroidism can cause cardiac dysrhythmias and osteoporosis. The clinical manifestations can also occur when there is excessive thyroid replacement.

What is the most common cause of gynecomastia? A. Puberty B. Drug use C. Testicular failure D. Malnutrition

A. Puberty

To reduce the incidence of flares, foods high in what amino acid need to be limited in the diets of patients with gout? A. Purine B. Glutamine C. Phenylalanine D. Alanine

A. Purine High-purine foods need to be avoided in order to reduce the risk of gout flares. High-purine foods include fish, red meat, and beans. Beer is also rich in purine.

A 35-year-old female presents to your primary care office for review of her laboratory results. Her physical exam shows a blood pressure (BP) of 140/90, pulse (P) of 105, oxygen saturation of 97%, and temperature of 98.6°F. She has complaints of palpitations, weight loss, hair loss, and anxiety. Her labs are all normal except for a low thyroid-stimulating hormone (TSH) and an elevated thyroxine (T4). What would your next course of treatment be? A. Start metoprolol and propylthiouracil (PTU) B. Refer the patient to psychiatry for treatment of anorexia C. Refer the patient for radioactive iodine treatment D. Refer the patient for thyroidectomy

A. Start metoprolol and propylthiouracil (PTU) This is the presentation of a patient with hyperthyroidism. A beta blocker would help treat her tachycardia and hypertension and PTU would help normalize her thyroid hormones.

Marty has pheochromocytoma. You instruct him to: a. void frequently in small amounts b. not exercise for > 30 minutes at a time. c. avoid sleeping in the prone position. d. take steroids.

A. Void frequently in small amounts Clients with pheochromocytoma should be told to void frequently in small amounts and to avoid a full bladder. In addition, to prevent stimulating a paroxysm, clients should also be advised to avoid smoking; drugs that may influence catecholamine release, such as some anesthetics, atropine, opiates, steroids, and glucagon; and activities that might displace abdominal organs, such as bending, exercising, straining, and vigorous palpation of the abdomen. For women, pregnancy should be discouraged.

Susie has diabetes mellitus. To forestall hypoglycemia, how much carbohydrate must she eat if she misses a regular meal? A. 5 to 15 g B. 15 to 30 g C. 30 to 35 g D. More than 35 g

Answer: B A client with type 1 diabetes mellitus who misses a meal needs to eat about 15 to 30 g of complex carbohydrates to forestall hypoglycemia. If the client's appetite is poor, this may be accomplished through juice, flavored gelatin, soft drinks, or frozen juice bars.

Jay has had diabetes for 10 years. He recently had a physical and was told he has some evidence of renal nephropathy. What is the first manifestation of this renal dysfunction? A. Microalbuminuria B. Development of Kimmelstiel-Wilson nodules C. Decreased blood urea nitrogen levels D. Increased serum creatinine levels

Answer: A Albuminuria is the first symptom indicative of renal nephropathy in clients who have had diabetes for about 10 years (although some studies suggest 5 years). There is increased permeability of the capillaries, with resultant leakage of albumin into the glomerular filtrate, causing albuminuria. The development of Kimmelstiel-Wilson nodules occurs in persons with type 1 diabetes but does not necessarily precede the albuminuria. As renal function deteriorates, both the serum creatinine and the blood urea nitrogen levels increase.

Which of the following individuals would not be a candidate for a thiazolidinedione such as pioglitazone (Actos)? A. A client with a history of hyperuricemia B. A client with asthma C. A client with heart failure D. A client with a sulfa allergy

Answer: C Thiazolidinediones, of which only pioglitazone (Actos) is available, cause or exacerbate heart failure. A Black Box Warning from the FDA is included in the prescribing information to warn providers.

Which of the following statements is true about insulin lispro (Humalog)? A. It works faster than regular insulin because its amino acid composition has been slightly modified. B. It should be injected 30 minutes before a meal. C. Its duration of action is about 6 to 8 hours. D. It is taken with the first bite of food.

Answer: A Insulin lispro is a rapid-acting human insulin whose amino acid composition has been slightly modified to make it work faster. It is administered 5 to 10 minutes before meals and reduces after-meal hyperglycemia to a greater extent than regular insulin. Its duration of action is about 3 hours, rather than the 5 to 6 hours of regular insulin. Because of this, lispro reduces the risk of hypoglycemia between meals and during the night

A 35-year-old client presents with a fasting plasma glucose (FPG) of 118 mg/dL and a HbA1c of 5.7%. Which of the following statements is true about these measurements? A. The client has prediabetes. B. The client has diabetes mellitus. C. The client has metabolic syndrome. D. The client has normal FPG and HbA1c

Answer: A The American Diabetes Association (ADA) defines prediabetes as a HbA1c of 5.7% to 6.4%. The American College of Clinical Endocrinologist defines prediabetes as a HbA1c of 5.5%. This patient has characteristics of metabolic syndrome; however, the term prediabetes is preferred in order to encourage early and aggressive therapeutic lifestyle changes (TLCs). The terms impaired fasting glucose and impaired glucose tolerance are also discouraged in favor of using the term prediabetes . Ideally, a trial of TLCs for 3 months will demonstrate an improvement. If you start pharmacologic treatment with metformin in the patient with prediabetes, you will not be able to determine if the improvement was due to TLCs or the metformin.

Eunice, age 32, has type 2 diabetes. She said that she heard that she should take an aspirin a day after she reaches menopause for its cardioprotective action. She does not have coronary artery disease, but her father does. How do you respond ? A. "You're right. Your hormones protect you against coronary artery diseases until menopause; then you should start on aspirin therapy." B. "The American Diabetes Association recommends that you start on low-dose aspirin therapy now." C. "Aspirin therapy is recommended for all patients over age 55 as a precautionary measure. D. "If you maintain good glycemic control, you don't need aspirin therapy."

Answer: A The American Diabetes Association's position statement on aspirin therapy in patients with diabetes recommends low-dose (81 mg) aspirin use as a secondary prevention strategy in men and women with diabetes who have evidence of large-vessel disease, such as a history of myocardial infarction, vascular bypass procedures, and stroke, and have no contraindications for the use of aspirin. They also recommend aspirin therapy as a primary prevention strategy in high-risk men and women with type 1 or type 2 diabetes who have a family history of coronary heart disease and for individuals who smoke, are hypertensive or obese, or who have albuminuria, cholesterol levels greater than 200 mg/dL, low-density lipoprotein cholesterol levels greater than 130 mg/dL, high-density lipoprotein cholesterol levels less than 40 mg/dL, and triglyceride levels greater than 250 mg/dL. Aspirin therapy is not recommended in all patients over age 55 as a precautionary measure.

A client with type 2 diabetes is on the maximum dosage of three oral antidiabetic agents, and the HbA1c remains at 8.5%. The practitioner initiates basal insulin. Which of the following would be an appropriate order? A. Insulin glargine (Lantus) 10 units nightly B. Insulin detemir (Levemir) 10 units before each meal C. Insulin aspart (NovoLog) 10 units before each meal D. Regular insulin before each meal

Answer: A The preference for basal insulins, such as glargine (Lantus) and detemir (Levemir), combined with mealtime or prandial rapid-acting insulin analogues, such as aspart (NovoLog), lispro (Humalog), and glulisine (Apidra), injected once a day, is based on the ability of this regime to closely mimic the normal kinetics and dynamics of endogenous insulin. This method, called basal-bolus, is easy to initiate, allows for flexible mealtimes, has less incidence of hypoglycemia, and helps clients reach the HbgA1c goal safely. The disadvantages of the insulin analogues are their increased price and the increased frequency of up to four injections a day. Neither detemir nor glargine can be mixed with other types of insulin. Many providers, especially endocrinologists, have abandoned the use of regular, neutral protamine Hagedorn and combinations such as Novolin 70/30. Basal insulin is usually 50% of the calculated insulin requirement in a day. Insulin requirements range from 0.4 units/kg per day to 0.7 units/kg per day. The basal dose is half of the total daily dose. The remaining amount can be divided and given as rapid-acting mealtime injections prior to breakfast, lunch, and dinner if needed. It is common to initiate basal insulin using glargine or detemir at 10 units at night (or 0.2 units/kg per day), letting the client know that additional injections will be required. The basal insulin can be adjusted by 2 units every 4 to 5 days until the fasting blood sugar is normalized. If the HbA1c remains elevated, the initiation of mealtime insulin with a rapid-acting analogue can be started with one injection with the largest meal. Many providers start with 4 units of a rapid-acting analogue such as aspart and have the client check blood sugar levels 2 hours after the start of the meal. If the postprandial glucose is elevated 2 hours after the start of the meal, the mealtime dosage can be increased by 1 to 2 units.

Martin, age 62, has acute nontransient abdominal pain that grows steadily worse in the epigastric area and radiates straight through to the back. The pain has lasted for days. He is also complaining of nausea, vomiting, sweating, weakness, and pallor. Physical examination reveals abdominal tenderness and distention and a low-grade fever. What do you suspect? A. Cholecystitis B. Acute pancreatitis C. Cirrhosis D. Cushing's syndrome

Answer: B Acute pancreatitis is an inflammation of the pancreas caused by the release of activated pancreatic enzymes into the surrounding parenchyma with the subsequent destruction of tissue, blood vessels, and supporting structures. Although pancreatitis may be acute or chronic, acute symptoms include continuous abdominal pain for several days' duration that increases in the epigastric area and radiates to the back, nausea, vomiting, sweating, weakness, pallor, abdominal tenderness, distention, and a low-grade fever. Pancreatitis occurs primarily in middle-aged adults and slightly more often in women than in men. The pain is in the upper right quadrant with cholecystitis and is intermittent, usually after a fatty meal. The gastrointestinal (GI) manifestations of cirrhosis include parotid enlargement, esophageal or rectal varices, peptic ulcers, and gastritis. The clinical manifestation of Cushing's syndrome related to the GI system is a peptic ulcer, which would result in intermittent pain related to meals.

One of the three Ps of diabetes is A. paresthesias. B. polydipsia. C. polycythemia. D. proteinuria.

Answer: B Although paresthesias (tingling or numbness in the hands or feet) may be one of the warning signs of diabetes, the classic symptoms remain the "Three Polys"—polyuria, polydipsia, and polyphagia. Proteinuria can also occur, although it is not one of the classic warning symptoms

Mark has type 1 diabetes and has mild hyperglycemia. What effect does physical activity (exercise) have on his blood glucose level? A. It may cause it to vary a little. B. It may decrease it. C. It may elevate it. D. It may fluctuate greatly either way.

Answer: B Clients with insulin-dependent diabetes mellitus (IDDM) type 1 who have mild hyperglycemia may experience a drop in their blood glucose level during physical activity, whereas those with marked hyperglycemia may experience a rise in their blood glucose level. Clients with IDDM should check their blood glucose level before exercising and refrain from exercising if their blood glucose levels are too high (greater than 300 mg). For individuals without diabetes, the blood glucose level generally varies little during physical activity unless the activity is intense and of very long duration, such as marathon running.

Betty, age 40, has had type 1 diabetes for 20 years and takes a combination of neutral protamine Hagedorn (NPH) and regular insulin every day. She comes to the office because she has developed a severe upper respiratory infection with chills, fever, and production of yellow sputum. Because of her acute infection, you know that Betty is likely to require A. a decrease in her daily insulin dosage. B. an increase in her daily insulin dosage. C. a high-caloric dietary intake and no insulin change. D. a change in her insulin from NPH to insulin aspart (NovoLog)

Answer: B For clients with diabetes requiring insulin, an increase in their daily insulin dosage is usually required in the presence of an acute infection. Betty should begin by increasing her regular insulin dosage by just 2 units and then monitoring her blood sugar level. At some point, a more physiologic insulin regime such as basal-bolus might be considered.

Morton has type 2 diabetes. His treatment, which includes diet, exercise, and three oral antidiabetic agents at maximum dose, is insufficient to achieve acceptable glycemic control. Your next course of action is to A. give the patient a sliding scale with mealtime coverage with regular insulin. B. add a dosage of long-acting insulin at bedtime to the regimen. C. discontinue the oral antidiabetic agents and start insulin therapy with N and R. D. suggest treatment using an insulin pump.

Answer: B If treatment with diet, exercise, and oral antidiabetic agents is insufficient to achieve acceptable glycemic control in clients with type 2 diabetes, adding a dose of insulin at bedtime to the regimen may be necessary. As a first step, the addition of a bedtime injection of long-acting insulin such as insulin glargine (Lantus) or insulin detemir (Levemir) is recommended. Intermediate-acting insulin such as neutral protamine Hagedorn (NPH) is no longer recommended because of the peaks in drug levels that can cause hypoglycemia. Initially, the dosage is 10 units at bedtime; then the dose is adjusted to reduce overnight hepatic glucose production and achieve a normal or near-normal fasting blood glucose concentration. If this regimen does not achieve the desired effect, the oral antidiabetic agents should be discontinued and mealtime analogue rapid-acting insulin can be added to the largest meal. Most clients will eventually require four injections with the basal-bolus regime. Insulin pumps are sometimes ordered for type 1 diabetes. As someone with type 2 diabetes, Morton should be able to regulate his insulin with the above actions.

The following is a self-monitoring blood glucose (SMBG) log on a client receiving 20 units Novolin 70/30 in the morning (a.m.) and 20 units Novolin 70/30 in the evening (p.m.): Fasting a.m.: 90, 98, 112, 100. Predinner: 140, 150, 105, 144. What changes would you make? A. Increase the p.m. insulin B. Increase the a.m. insulin C. Add insulin aspart (NovoLog) at bedtime. D. Increase the a.m. and p.m. insulin.

Answer: B Insulin mixes such as NovoLog 70/30 and Novolin 70/30 were developed to deliver a more convenient medication regime. Seventy percent of the mix is NPH, and 30% is either regular or a rapid-acting insulin analogue. It is given twice a day before breakfast and dinner. If the rapid-acting portion is regular insulin, it should be injected 30 minutes before the meal. If the rapid-acting portion is insulin aspart (NovoLog), it can be injected 5 to 15 minutes before the meal. The insulin mixes are a less flexible dosing pattern than basal-bolus because the client is required to eat midday or will suffer hypoglycemia. In this case, the morning fasting levels are normal, and the predinner glucose levels are elevated. Increasing the morning dose by 2 units will lower the predinner glucose levels. The regular insulin will also be increased and can lead to low blood sugar at lunchtime. Many providers are getting away from the insulin mixes because of the problems with hypoglycemia and are substituting the more flexible basal-bolus insulin regimes.

Pancreatitis has been associated with which of the following antidiabetic drugs ? A. Glipizide (Glucotrol) B. Metformin (Glucophage) C. Insulin glargine (Lantus) D. Exenatide (Byetta)

Answer: D The incretin-like drugs are oral agents, such as the dipeptidyl-peptidase-4 (DPP-4) inhibitors (e.g., sitagliptin [Januvia] and saxagliptin [Onglyza]), and injectable incretin mimetics or analogues, such as exenatide (Byetta) and liraglutide (Victoza). All of these agents have been associated with increased risk of pancreatitis and should not be used in clients with a history of pancreatitis. Clients and providers should be aware of the signs and symptoms of pancreatitis. Victoza has been associated with and increased risk of medullary thyroid cancer.

Marie, age 50, has type 1 diabetes and checks her blood glucose level several times every day. Her blood glucose level ranges from 250 to 280 mg/dL in the morning and is usually about 140 at lunch, about 120 at dinner, and about 100 at bedtime. In the morning she takes 30 units of neutral protamine Hagedorn (NPH) insulin and 4 units of regular insulin, and before dinner she takes 18 units of NPH insulin and 4 units of regular insulin. Although she has had her insulin dosage adjusted several times in the past month, it has had no effect on her high morning blood glucose level. What is your next course of action? A. Increase the evening NPH insulin dosage by two more units. B. Have her check her blood glucose level between 2 a.m. and 4 a.m. for the next several days. C. Increase the morning regular insulin dosage by two units. D. Order a fasting blood sugar test.

Answer: B Marie is experiencing the Somogyi phenomenon (nocturnal rebound hyperglycemia). If her blood glucose level at 2 a.m. to 4 a.m. is greater than 70 mg/dL, the evening dosage of neutral protamine Hagedorn (NPH) insulin should be increased and changed from before dinner to before bedtime. These actions should prevent most cases of nocturnal rebound hypoglycemia, which results in morning hyperglycemia. A fasting blood sugar test will not confi rm the Somogyi phenomenon. The blood sugar level needs to be checked during the night to "catch" the Somogyi phenomenon. Many providers prefer the longer-acting insulins such as insulin glargine (Lantus) and insulin detemir (Levemir) because they are mostly "peakless" and have less risk of hypoglycemia than NPH.

Morris has had type 1 diabetes for 10 years. Several recent urinalysis reports have shown microalbuminuria. Your next step would be to A. order a 24-hour urinalysis. B. start him on an angiotensin-converting enzyme (ACE) inhibitor. C. stress the importance of strict blood sugar control. D. send him to a dietitian because he obviously has not been following his diet.

Answer: B Morris should be started on an ACE inhibitor such as enalapril (Vasotec). ACE inhibitors have renoprotective effects by reducing the intraglomerular pressure. They do this by inhibiting the renin-angiotensin system, which causes efferent dilation, and by improving glomerular permeability, which causes a reduction of glomerulosclerosis. ACE inhibitors also have this benefi cial effect on clients with diabetes who are normotensive and even hypotensive. Diabetic nephropathy is the leading cause of end-stage renal disease in the United States. Monitoring for microalbuminuria is a method for identifying early nephropathy. Ordering a 24-hour urinalysis will not give you any additional information. You do want to stress tight glycemic control and possibly send Morris to a dietitian, but he needs to be started on an ACE inhibitor now because he is already exhibiting microalbuminuria.

Older clients with diabetes are predisposed to which of the following types of ear infections? A. Simple otitis externa B. Malignant otitis externa C. Otitis media D. Serous otitis media

Answer: B Older clients with diabetes are predisposed to malignant otitis externa, caused by Pseudomonas aeruginosa. It is an invasive and necrotizing infection with a high mortality, mainly because of meningitis. The client complains of pain in the ear with or without a purulent drainage, swelling of the parotid gland, trismus (tonic contraction of the muscles of mastication), and paralysis of the 6th through 12th cranial nerves. The other types of ear infections may happen at any time, but these patients are predisposed to malignant otitis externa.

Polydipsia occurs in diabetes as a result of a high serum glucose level, which A. interferes with the release of antidiuretic hormones. B. has an osmotic effect on fluids and eventually triggers the thirst mechanism for compensation. C. causes a dry mouth, increasing the client's thirst to the point of drinking compulsively. D. disrupts fluid and electrolyte imbalance, increasing the thirst mechanism to compensate for fluid loss or gain.

Answer: B Polydipsia occurs in diabetes as a result of high serum glucose levels that have an osmotic effect on fluids, drawing fluid from the cells into the intravascular space and creating a dehydrated state that, in turn, triggers the thirst mechanism for compensation. With a deficiency of antidiuretic hormone, as in diabetes insipidus, copious amounts of urine are excreted by the client, stimulating the thirst mechanism to replace fl uid losses. Certain drugs such as phenothiazines and anticholinergics can cause dry mouth, increasing the client's thirst to the point of drinking compulsively. Excessive ingestion of salt, glucose, and other hyperosmolar substances can disrupt fl uid and electrolyte imbalance, increasing the thirst mechanism to compensate for fluid loss or gain.

Sandra, age 28, has secondary obesity. Which of the following may have caused this? A. An intake of more calories than are expended B. Polycystic ovary disease C. Her antihypertensive medications D. Her sedentary lifestyle

Answer: B Secondary obesity is rare; possible causes include Cushing's disease, polycystic ovary disease, hypothalamic disease, hypothyroidism, and insulinoma. Some medications associated with weight gain include glucocorticoids, tricyclic antidepressants, and phenothiazines. Essential obesity is the most prevalent type and is the result of the intake of more calories than are expended. This type of obesity results from the multiple interactions of genetic and environmental factors (cultural, metabolic, social, and psychological). Antihypertensive medications do not lead to obesity. A sedentary lifestyle may lead to primary, not secondary, obesity.

Harriet, age 62, has type 1 diabetes that is well controlled by insulin. Recently, she has been having marital difficulties that have left her emotionally upset. As a result of this stress, it is possible that she will A. have an insulin reaction more readily than usual. B. have an increased blood sugar level. C. need less daily insulin. D. need more carbohydrates.

Answer: B Stress causes the adrenal glands to secrete more cortisol, which leads to gluconeogenesis and insulin antagonism, raising the blood sugar. It is possible, then, that Harriet will have an increased blood sugar level. She will not need less daily insulin or more carbohydrates and will not have an insulin reaction, such as hypoglycemia, more readily than usual. Harriet may, in fact, need to increase her insulin use.

The process of aging results in A. an increase in liver weight and mass. B. a decreased absorption of fat-soluble vitamins. C. an increase in enzyme activity. D. constricted pancreatic ducts.

Answer: B The process of aging results in a decreased absorption of fat-soluble vitamins. There is a decrease in the number and size of hepatic cells, leading to a decrease in liver weight and mass. There is also a decrease in enzyme activity, which diminishes the liver's ability to detoxify drugs, which increases the risk of toxic levels of many medications in older adults. There is calcification of the pancreatic vessels, and the ducts distend and dilate. These changes lead to a decrease in the production of lipase.

A client with diabetes complains of paresthesias. Which of the following medications can cause sensory changes ? A. Glipizide B. Sitagliptin (Januvia) C. Metformin D. Exenatide (Byetta)

Answer: C A rare adverse effect of metformin is megaloblastic anemia due to impaired absorption of vitamin B 12. This can be prevented by administration of vitamin B12 along with folic acid. Nerve degeneration often begins early in the course of diabetes and can also cause paresthesias. Nerve damage is directly related to sustained hyperglycemia, which may cause metabolic disturbances in nerves or may injure the capillaries that supply nerves. Diabetic neuropathy necessitates vigilance in the care of the extremities to prevent damage. Sensory changes are not listed as adverse reactions to the other medication choices.

A client with diabetes presents with a medication list that includes lisinopril and losartan. Which of the following statements is true regarding this combination? A. Additional blood pressure and renal benefit is gained using an ACE inhibitor and ARB. B. This combination will lead to excessive lowering of the potassium. C. This was probably a provider error or patient misunderstanding. D. Combination therapy is recommended for patients with hypertension.

Answer: C ACE inhibitors and angiotensin receptor blockers should not be used together. This is likely a misunderstanding by the patient or error by the provider. The combination of an ACE inhibitor and ARB offers no additional renal benefit; rather, it causes excessive blood pressure lowering and risk of hyperkalemia.

ACE inhibitors are given to clients with diabetes who have A. an elevated glycohemoglobin level. B. insulin sensitivity. C. persistent albuminuria. D. an elevated serum creatinine level

Answer: C ACE inhibitors are given to clients with diabetes who have persistent albuminuria. When diabetes is diagnosed, it may have been present for 10 years. Microalbumin measurements of greater than 30 mg per day justifies the addition of an ACE inhibitor in normo-tensive clients. Albuminuria is one of the early signs of diabetic nephropathy, with or without the presence of hypertension. ACE inhibitors may be effective in decreasing urinary excretion of albumin even in clients without hypertension; therefore, a urinalysis should be performed, and a serum creatinine level should be determined at least yearly. Detection of microalbuminuria alerts one that nephropathy is developing. When the serum creatinine level reaches about 3 mg/dL, a referral to a diabetologist and nephrologist should be done. Intensive treatment can delay or improve diabetic nephropathy in its early stages. There is evidence that, with good control of hypertension, albuminuria can be reduced and the expected decline in the glomerular fi ltration rate can be slowed.

The American Diabetes Association (ADA) recommends which of the following quarterly blood tests to be performed on all clients with diabetes? A. TSH B. Liver function C. Glycohemoglobin D. Serum glucose

Answer: C Although a serum glucose test is an excellent test for clients with diabetes, it reports only the serum glucose of that day. The American Diabetes Association (ADA) therefore recommends that the glycohemoglobin (hemoglobin A 1C ) test be performed quarterly because it reports the serum glucose concentration of the previous 3 months. HbA1c can now be used for diagnosis of diabetes (greater than 6.5%). The ADA also recommends an annual urine test to assess for urine protein that might indicate an early sign of kidney damage. Liver function studies should be done on an annual basis as part of a routine examination. The American Thyroid Association recommends measuring thyroid function in all adults beginning at age 35 and every five years thereafter.

Mason, age 52, has diabetes and is overweight. You now find that he is hypertensive. How should you treat his hypertension? A. You should treat it the same as in a client without diabetes. B. Because insulin affects most of the antihypertensive drugs, you should try diet and exercise first before any antihypertensives are ordered. C. You should treat it very aggressively, preferably with ACE inhibitors. D. You should initiate therapy when the blood pressure is 5 to 10 mm Hg more than the conventional therapeutic guidelines.

Answer: C Because hypertension is implicated in accelerating the microangiopathy of diabetes (especially retinopathy and nephropathy), therapy should be initiated when the patient has a blood pressure at or above 140/90 mm Hg according to JNC 8. Other organizations recommend the lower goal of 130/80 in clients with DM. ACE inhibitors are the drugs of choice in clients with DM with hypertension owing to the benefi cial effects of reducing albuminuria and reduction of glomerular pressure. ACE inhibitors should be initiated in patients with diabetes and an elevated blood pressure and are recommended in normotensive patients with albuminuria.

Which class of antihypertensive agents may be problematic for clients with diabetes? A. ACE inhibitors B. Calcium channel blockers C. Beta blockers D. Alpha blockers

Answer: C Beta blockers may be problematic in clients with diabetes because they block the first sign of hypoglycemia, which is often tachycardia. Many clients with diabetes have compelling indications for the use of beta blockers, such as coronary artery disease. In these clients the need for a beta blocker outweighs any risk that occurs. Decreasing the possibility of low blood sugar by selecting appropriate agents and adjusting dosages may be necessary. If the client with diabetes is on a beta blocker, it is important to explain that instead of the tachycardia, he or she will notice other signs of hypoglycemia, such as sweating, that will still occur if the client is taking a beta blocker. ACE inhibitors are the first choice for clients with diabetes who have hypertension because they slow the progression of diabetic nephropathy. Calcium channel blockers provide pressure reduction without adverse effects on lipids and glucose control. Alpha-blocking agents provide a smooth control and an improved lipid profi e.

Mindy is scheduled to have an oral glucose tolerance test. For 3 days before the test, she is instructed to discontinue many of her medications. Which one is it safe to continue taking? A. Vitamin C B. Aspirin C. Calcium D. Her oral contraceptive

Answer: C Calcium does not affect an oral glucose tolerance test (OGTT). The following medications may interfere with the results of an OGTT and should be discontinued for 3 days before the test: vitamin C, aspirin, oral contraceptives, corticosteroids, synthetic estrogens, phenytoin (Dilantin), thiazide diuretics, and nicotinic acid. For clients taking oral contraceptives, be sure to recommend they use another form of birth control for this period of time

When teaching Marcy how to use her new insulin pump, you tell her that she needs to monitor her blood glucose level A. at least once a day. B. only occasionally because glycemic levels are maintained very steadily. C. at least four times a day. D. on an as-needed basis when she feels she needs to give herself an extra dose of insulin.

Answer: C Clients using an insulin pump need to monitor their blood glucose levels at least four times a day. The client can develop diabetic ketoacidosis in as little as 4 hours if there is mechanical failure of the pump because the only insulin used in the pump is rapid acting.

Pathological changes that occur with diabetic neuropathies include A. a thinning of the walls of the blood vessels that supply nerves. B. the formation and accumulation of amino glycosyl within the Schwann cells, which impairs nerve conduction. C. demyelinization of the Schwann cells, which results in slowed nerve conduction. D. increase of nutrients clogging the vessels that supply nerve endings.

Answer: C Diabetic neuropathies involve three pathological changes: a thickening of the walls of the blood vessels that supply nerves, which causes a decrease in nutrients to the nerves; the formation and accumulation of sorbitol within the Schwann cells, which impairs nerve conduction; and demyelinization of the Schwann cells that surround and insulate nerves, which results in slowed nerve conduction. The locations of the lesions determine where the neuropathies occur.

Tamika, who has diabetes, states that she heard that fi =ber is especially good to include in her diet. How do you respond? A. "Fiber is important in all diets." B. "Too much fiber interferes with insulin, so include only a moderate amount in your diet." C. "Fiber, especially soluble fiber, helps improve carbohydrate metabolism, so it is more important in the diet of persons with diabetes." D. "You get just the amount of fiber you need with a normal diet.

Answer: C Fiber is important in the dietary management of diabetes. A diet high in fiber, especially soluble fiber, helps improve carbohydrate metabolism, lowers total cholesterol level, and lowers low-density lipoprotein cholesterol. Soluble fiber is found in dried beans, oats, barley, and in some vegetables and fruits (peas, corn, zucchini, caulifl ower, broccoli, prunes, pears, apples, bananas, and oranges). It should not be assumed that individuals will get enough fiber in their diet because most dietary habits are not perfect. An intake of 20 to 30 g of fiber per day is recommended. While fiber is important in all diets, answer A does not address why fiber is especially good to include in the diet of a patient with diabetes.

Jenny, age 46, has hypertension that has been controlled with hydrochlorothiazide 50 mg every day for the past 3 years. She is 5 ft 8 in. tall and weighs 220 lb. Her fasting blood sugar (FBS) level is 300 mg/dL, serum cholesterol level is 250 mg/dL, serum potassium level is 3.4 mEq, and she has 4+glucosuria. Your next course of action would be to A. discontinue her hydrochlorothiazide. B. order a glucose tolerance test (GTT). C. repeat her FBS test and do an HbA1c. D. start insulin therapy.

Answer: C Jenny's fasting blood sugar (FBS) test should be repeated along with an HbA1c. An HbA1c of greater than 6.5 can now be used to diagnosis diabetes. A glucose tolerance test (GTT) to confirm a diagnosis of diabetes is usually not needed. Diabetes is not usually diagnosed with a single high glucose reading unless symptoms of polyphagia, polydipsia, and polyuria are present. Hyperglycemia can be an adverse reaction to high doses of hydrochlorothiazide, but the first action would be to repeat the FBS. If it is high for a second reading, the diuretic should be reduced. Insulin therapy would not be started until Jenny was given a positive diagnosis; and even then, oral antidiabetic agents would be considered first.

Your client with diabetes asks you about insulin glargine (Lantus). You tell her that A. it may be administered subcutaneously at home or intravenously in the hospital if need be. B. the onset of action is 15 minutes. C. insulin glargine (Lantus) stays in your system for 24 hours. D. it can be mixed with any other insulin.

Answer: C Lantus (insulin glargine) has an onset of action in just over 1 hour and stays in the system for 24 hours. Regular insulin may be administered by the intravenous route. The newer insulin analogues such as insulin aspart (Novolog) are also approved for intravenous use. Because of the cost, there is little reason to use the more expensive agents. Insulin glargine (Lantus) and insulin detemir (Levemir) may not be mixed with any other insulin.

Leah, age 70, has had diabetes for many years. When teaching her about foot care, you want to stress A. that her calluses will protect her from infection. B. the need to assess the bottom of her feet carefully after walking barefoot. C. that painless ulceration might occur and feet should be examined with a mirror. D. that mild pain is to be expected because of neuropathies.

Answer: C Painless ulcerations are very common in clients with diabetes, and the only way to assess for them in the feet is for clients to use a mirror to examine the bottoms of their feet. Leah should not be walking barefoot because her sensations are probably decreased as a result of neuropathy, and she should try to avoid the development of calluses because preparations used to remove them are very caustic.

Which type of insulin would be expected to have the earliest onset of action? A. NPH B. Insulin glargine (Lantus) C. Insulin aspart (NovoLog) D. Novolin-R

Answer: C The rapid-acting insulin analogues (such as LAG = lispro, aspart, and glulisine) are preferred because they mimic endogenous secretion of insulin. When an individual with a normal functioning pancreas eats a meal, the insulin secretion is immediate. The newer analogue insulins have a very rapid onset of action of 5 to 15 minutes, which can match the increase in blood sugar associated with a meal. These "physiologic" insulins are more efficacious and provide greater flexibility for those clients with variable mealtimes and carbohydrate content of meals. The duration of action of the rapid-acting insulin analogues is shorter than regular insulin, and the chances of hypoglycemia are less.

Diabetes and coronary artery disease (CAD) have a close interrelationship. Which of the following statements about the relationship between diabetes and CAD is true? A. Hyperinsulinemia decreases sympathetic tone and cardiac contractility by increasing plasma catecholamines, epinephrine, and norepinephrine. B. An increase of glucose causes the distal nephrons of the kidneys to absorb less sodium, resulting in more fluid, expanding the intravascular volume, and increasing the blood pressure. C. Hyperinsulinemia causes a large number of vascular smooth muscle cells to be formed and deposited on the walls of vessels, eventually decreasing space for blood fl ow. D. An increase in glucose stimulates more secretion of epinephrine, thus raising the blood pressure (BP).

Answer: C There is a major link between diabetes mellitus and coronary artery disease (CAD). There are many theories about the pathophysiology of diabetes mellitus and how it leads to CAD. Some of these mechanisms include the following: Hyperinsulinemia causes a large number of vascular smooth muscle cells to be formed and deposited on the walls of vessels, causing buildup and eventual blockage, reducing blood flow; hyperinsulinemia increases (not decreases) sympathetic tone and cardiac contractility by increasing plasma catecholamine, epinephrine, and norepinephrine levels; and an increase in the glucose level causes the distal nephrons of the kidneys to absorb more (not less) sodium, resulting in more fl uid, expanding the intravascular volume and increasing blood pressure.

Mandy has type 2 diabetes. She says she heard that if she becomes pregnant, she must go on insulin therapy. How do you respond? A. "You're under good glycemic control now with your oral agents. As long as you stay in good control, you'll stay on the medication you are on now." B. "Don't worry about it now; wait until you get pregnant." C. "Insulin is commonly used during pregnancy. Some oral agents are now being used." D. "You need to start on insulin therapy now before you get pregnant. You'll also need it throughout your pregnancy."

Answer: C Women with type 2 diabetes often need to use insulin during their pregnancy to maintain good glycemic control. There is a risk of having a malformed infant if conception occurs when the blood sugar is not well controlled. Insulin, rather than oral antidiabetic medications, is often used to achieve good glycemic control because the effects of most of the oral antidiabetic preparations on the human embryo and fetus are not well known. Metformin appears to be safe in pregnancy and is now being used by some providers. Many clients do not require antidiabetic therapy for the first several weeks after delivery, and most women can return to their preconception regimen within 6 to 12 weeks. Sulfonylureas may be transferred to the fetus via breast milk, so they should not be resumed until breastfeeding is discontinued.

After an oral cholecystogram, Sam complains of burning on urination. This is because of A. a mild reaction to the contrast medium. B. biliary obstruction. C. contraction of the gallbladder. D. the presence of dye in the urine

Answer: D After an oral cholecystogram, some persons experience burning on urination because of the presence of dye in the urine. This is helped by forcing fluids. An oral cholecystogram is done to assess for biliary obstruction. Contraction of the gallbladder may be desirable during a cholecystogram to obtain a better reading and may be accomplished by having the client consume a high-fat meal during the procedure. A reaction to the contrast medium would produce symptoms such as urticaria, nausea, vomiting, or dyspnea.

Sara, age, 40, has diabetes and is now experiencing anhidrosis on the hands and feet, increased sweating on the face and trunk, dysphagia, anorexia, and heartburn. Which complication of diabetes do you suspect? A. Macrocirculation changes B. Microcirculation changes C. Peripheral neuropathies D. Autonomic neuropathies

Answer: D Autonomic neuropathies include anhidrosis (absence of sweating) on the hands and feet, increased sweating on the face or trunk, dysphagia, anorexia, heartburn, constricted pupils, nausea and vomiting, constipation, and diabetic diarrhea. Macrocirculation changes include an early onset of atherosclerosis and peripheral vascular insufficiency with claudication, ulcerations, and gangrene of the legs. Microcirculation changes include diabetic retinopathy with retinal ischemia and loss of vision and diabetic nephropathy with hypertension, albuminuria, edema, and progressive renal failure. Peripheral neuropathies include changes in sensation in the feet and hands; palsy of cranial nerve III with headache, eye pain, and inability to move the eye up, down, or to the middle; pain or loss of cutaneous sensation over the chest; and motor and sensory deficits in the anterior thigh and medial calf.

Jennifer has DM and is injecting 30 units of Novolin 70/30 with breakfast and 18 units at bedtime. She is complaining that she woke up once in the middle of the night with palpitations and sweating. Based on this information, what do you recommend ? A. Decrease the a.m. dose of 70/30. B. Decrease the p.m. dose of 70/30. C. Eat a snack before going to bed. D. Change the time of the nighttime insulin injection.

Answer: D The Novolin 70/30 mix should be given before a meal. Jennifer was not given appropriate instructions on the timing of the injections. She should inject her nighttime dose about 30 minutes prior to the evening meal. Injecting this type of insulin before bed will cause the blood sugar to drop because 30% of the insulin is regular insulin. NPH alone is not used much anymore due to the advent of the longer-acting insulin preparations. If it is used, it is common for it to be administered every 12 hours with the nighttime dose before bed. Changing the morning dose of insulin will not affect her symptoms. While decreasing her nighttime dose and eating a snack before bedtime might have some effect, the Novolin 70/30 must be given 30 minutes prior to the evening meal, not at bedtime.

A newly diagnosed client with diabetes who has an HbA1c of 7.5 is started on therapeutic lifestyle changes (TLCs) and medical nutritional therapy (MNT). Which oral antidiabetic agent is recommended as monotherapy? A. Glipizide (Glucotrol) B. Sitagliptin (Januvia) C. Exenatide (Byetta) D. Metformin (Glucophage)

Answer: D Because of its safety, efficacy, and cost, metformin is the cornerstone of monotherapy unless there is a contraindication, such as renal disease, hepatic disease, gastrointestinal intolerance, or risk of lactic acidosis. Metformin often has benefi cial effects on components of the metabolic syndrome, including mild to moderate weight loss, improvement of the lipid profile, and improved fibrinolysis. It improves the effectiveness of insulin in suppressing excess hepatic glucose production and increases insulin sensitivity in peripheral tissues. Hypoglycemia with the use of metformin is low. Gastrointestinal side effects can be diminished by starting at the lowest dose of 500 mg daily and gradually increasing the dosage as needed to a maximum dose of 2,000 mg per day. Use of metformin and alcohol can increase the risk of lactic acidosis. Insulin secretagogues such as sulfonylureas can cause hypoglycemia and are often added as a second, cost-effective choice. The dipeptidyl-peptidase-4 (DPP-4) inhibitors such as sitagliptin (Januvia) increase insulin secretion, suppress glucagon secretion, and suppress hepatic glucose production and peripheral glucose uptake and metabolism. An injectable agent, exenatide (Byetta), is an incretin mimetic that often induces weight loss. Both sitagliptin and exenatide have a lower risk of hypoglycemia, making them a good choice over a sulfonylurea as dual therapy. The cost of both of these newer agents may be prohibitive for many clients, and evidence for long-term reduction of morbidity and mortality is lacking. There is also a risk of pancreatitis with the oral and injectable incretin-like drugs.

Which of the following steps will not prevent or slow the progression of diabetic nephropathy? A. Control of blood pressure B. Use of ACE inhibitors C. Smoking cessation D. Use of beta adrenergic antagonists

Answer: D Beta blockers do not slow the progression of diabetic nephropathy in clients with diabetes. The steps that can be taken to avoid or slow the progression of diabetic nephropathy include maintaining excellent blood pressure control, using ACE inhibitors, smoking cessation, lowering cholesterol, and maintaining excellent glucose control.

Clients with diabetes are more prone to cardiovascular disease than those without diabetes. This is probably because A. of their difficulty in metabolizing fats and proteins, the end products of which accumulate in the blood vessels. B. they are usually overweight, which increases the workload on the heart and blood vessels. C. most are older adults, who are more likely to have degenerative cardiovascular disease. D. the high levels of glucose and fat that occur with poor control result in atherosclerotic changes in the blood vessels.

Answer: D Clients with diabetes are more prone to cardiovascular disease than those without diabetes. The most likely reason for this is that the high levels of glucose and fat that occur with poor control result in atherosclerotic changes in the blood vessels. The client with diabetes may also be overweight, but that is not the primary contributing factor to the development of cardiovascular disease. Diabetes is associated with a greater incidence of high blood lipid levels, high blood pressure, and obesity, all of which are risk factors for cardiovascular disease. Diabetes affects both small and large blood vessels, which contributes to the process of atherosclerosis. In women, diabetes negates the protective effects of estrogen. The amount of protein in the diet of a client with diabetes is restricted to help prevent or delay renal complications, not cardiovascular ones. The reason clients with diabetes, especially older adults, are more prone to cardiovascular disease than those without diabetes is the high levels of glucose and fat, which result in atherosclerotic changes in the blood vessels.

Clients with diabetes and neuropathy require special foot care because most ulcers begin at the site of A. an ingrown toenail. B. a previous sore. C. a mole. D. a callus.

Answer: D Clients with neuropathy require professional nail and callus care because most ulcers begin at the site of a callus. While an ulcer may occur at any site, this is the most common.

A client with diabetes on a sulfonylurea and metformin with an HbA1c of 6.5% is complaining of episodes of low blood sugar. Which of the following changes would be the most appropriate? A. Decrease the dosage of the metformin B. Discontinue the metformin C. Increase carbohydrate intake D. Decrease the dosage of the sulfonylurea.

Answer: D Metformin, DPP-4 inhibitors such as sitagliptin (Januvia), and incretin mimetics such as exenatide (Byetta) are gaining favor over sulfonylureas because the risk of hypoglycemia is less than with sulfonylureas. Increasing the carbohydrate intake is never a good choice in a patient with diabetes.

Ben, a client with type 1 diabetes, is hospitalized with an admitting diagnosis of diabetic ketoacidosis. Which of the following signs and symptoms would be consistent with this condition? A. Hypoglycemia and glycosuria B. Decreased respiratory rate with shallow respirations C. Polydipsia and an increased blood pH D. Ketonuria and polyuria

Answer: D Signs and symptoms of diabetic ketoacidosis include Kussmaul's breathing (very deep respiratory movements), hyperglycemia, glycosuria, polyuria, polydipsia, anorexia, and headache, as well as ketonuria and a decreased blood pH. Hyperglycemia, not hypoglycemia, is a symptom of DKA, as are very deep, not shallow, respirations and a decreased, not increased, blood pH.

Which of the following supports a diagnosis of diabetes mellitus? A. Fasting plasma glucose is higher than 100 mg/dL B. Two-hour plasma glucose is 150 mg/dl or higher (by OGTT with a 75-g glucose load) C. Patient has classic symptoms of hyperglycemia or hyperglycemic crisis with a random plasma glucose of 150 mg/dL or higher D. Hemoglobin A1c is 6.5% or higher

Answer: D The American Diabetes Association recommends screening for type 2 diabetes annually in patients age 45 and older and in patients younger than 45 with major risk factors. The diagnosis can be made with a fasting plasma glucose level of 126 mg per dL or greater; an A1C level of 6.5% or greater; a random plasma glucose level of 200 mg per dL or greater; or a 75-g, 2-hour oral glucose tolerance test with a plasma glucose level of 200 mg per dL or greater. Results should be confirmed with repeat testing on a subsequent day; however, a single random plasma glucose level of 200 mg per dL or greater with typical signs and symptoms of hyperglycemia likely indicates diabetes.

Early-morning increases in blood glucose concentration that occur with no corresponding hypoglycemia during the night are referred to as A. the Somogyi phenomenon. B. insulin shock. C. diabetic ketoacidosis. D. the dawn phenomenon.

Answer: D The dawn phenomenon is an early-morning increase in blood glucose concentration occurring with no corresponding hypoglycemia during the night. It is secondary to the nocturnal elevations of growth hormone. The Somogyi phenomenon is a rebound effect caused by too much insulin at night, resulting in hypoglycemia during the night. This results in the secretion of certain anti-insulin hormones, including epinephrine, glucagon, glucocorticoids, and growth hormones, which results in hyperglycemia. Insulin shock is a hypoglycemic reaction. Diabetic ketoacidosis implies that there is hyperglycemia at all times during the day, not only in the early morning.

Mary, age 72, has been taking insulin for several years. She just called you because she realized that yesterday she put her short-acting insulin in the long-acting insulin box and vice versa. She just took 22 units of regular insulin when she was supposed to take only 5 units. She says that she tried to do a fingerstick to test her glucose level but was unable to obtain any blood. She states that she feels fine. What do you tell her to do first? A. "Keep trying to get a fingerstick and call me back with the results." B. "Call 911 before you collapse." C. "Drive immediately to the emergency room." D. "Drink 4 oz of fruit juice."

Answer: D Treatment of hypoglycemia is 15 grams of carbohydrates and can be achieved with drinking 8 oz of milk or 4 oz of orange juice. The milk option is preferred to decrease the elevation in blood sugar that occurs with treatment of hypoglycemia with orange juice. The patient should wait for 15 minutes to see if the symptoms subside and repeat the treatment again if needed. All patients on insulin should be prescribed the Glucagon pen, and family or friends should be instructed on its use. Mary should eventually be able to get a finger stick but does not need to call if it is okay. The fruit juice will probably correct the problem, so she may not need to call 911. If her blood sugar is low, she should certainly not drive herself to the emergency room.

Dan, age 45, is obese and has type 2 diabetes. He has been having trouble getting his glycohemoglobin under control. He has heard that exenatide (Byetta) causes weight loss and wants to try it. What do you tell him? A. "Let's adjust your oral antidiabetic agents instead." B. "That's a myth. People usually change their eating habits when taking this, and that's what causes the weight loss." C. "With type 2 diabetes, you never want to be on injectable insulin." D. "Let's try it. You're glycohemoglobin will be lowered and you may lose weight."

Answer: D Unlike many oral antidiabetic agents, injectable exenatide (Byetta) can cause weight loss in some individuals. The active ingredient is a protein that encourages digestion and the production of insulin. The GLP-1 (glucagon-like peptide) injectables like exenatide and liraglutide (Victoza) may be used as an adjunct to diet and exercise to improve glycemic control in adults with type 2 diabetes mellitus. Adjusting the client's oral antidiabetic agents may not be as effective, but they are less expensive. They are not injectable insulins.

Sandra, age 31, has diabetes and frequently develops vaginitis. You instruct her to A. wipe from back to front after voiding. B. wear white nylon underwear. C. wear pantyhose if wearing tight jeans. D. avoid douching.

Answer: D Women who have diabetes and frequently develop vaginitis should avoid douching, as should all women. They should also maintain good personal hygiene, wipe from front to back after voiding, wear cotton underwear, avoid wearing tight jeans with nylon pantyhose, and void after intercourse.

If a client has hepatitis, abdominal pain in the right upper quadrant is a result of what pathophysiological basis? A. Reduced prothrombin synthesis by injured hepatic cells B. Bile salt accumulation C. Release of pyrogens D. Stretching of Glisson's capsule

Answer: D For the client with hepatitis, abdominal pain in the right upper quadrant is caused by stretching of Glisson's capsule surrounding the liver, which occurs because of inflammation; bleeding tendencies are a result of reduced prothrombin synthesis by injured hepatic cells, pruritus is caused by bile salt accumulation in the skin, and fever is the result of the release of pyrogens in the inflammatory process.

The major risk factor for thyroid cancer is: A. Inadequate iodine intake B. Presence of a goiter C. Exposure to radiation D. Smoking

C. Exposure to radiation The major risk factor for thyroid cancer is exposure to radiation, usually from treatment to the head and neck. Until 1950, radiation treatments were given to children for an enlarged thymus, enlarged tonsils, and acne. Several million children were exposed in this manner. It may also occur in individuals who have had radiation therapy to the face or upper chest.

The process of aging results in: A. An increase in liver weight and mass B. A decreased absorption of fat-soluble vitamins C. An increase in enzyme activity D. Constricted pancreatic ducts

B. A decreased absorption of fat-soluble vitamins Option 1: There is a decrease in the number and size of hepatic cells, leading to a decrease in liver weight and mass. Option 2: The process of aging results in a decreased absorption of fat-soluble vitamins. Option 3: There is a decrease in enzyme activity, which diminishes the liver's ability to detoxify drugs. This increases the risk of toxic levels of many medications in older adults. Option 4: There is calcification of the pancreatic vessels, and the ducts distend and dilate. These changes lead to a decrease in the production of lipase.

Morton has Type 2 diabetes. His treatment, which includes diet, exercise, and oral antidiabetic agents, is insufficient to achieve acceptable glycemic control. Your next course of action is to: A. Give the patient a sliding scale with mealtime coverage with regular insulin B. Add a dosage of long-acting insulin at bedtime to the regimen C. Discontinue the oral antidiabetic agents and start insulin therapy with N and R D. Suggest treatment using an insulin pump

B. Add a dosage of long-acting insulin at bedtime to the regimen If treatment with diet, exercise, and oral antidiabetic agents is insufficient to achieve acceptable glycemic control in clients with type 2 diabetes, adding a dosage of insulin at bedtime to the regimen may be necessary. As a first step, the addition of a bedtime injection of long-acting insulin such as insulin glargine (Lantus) or insulin detemir (Levemir) is recommended. Intermediate-acting insulin such as neutral protamine Hagedorn (NPH) is no longer recommended because of the peaks in drug levels that can cause hypoglycemia. Initially, the dosage is 10 units at bedtime; then the dose is adjusted to reduce overnight hepatic glucose production and achieve a normal or near-normal fasting blood glucose concentration. If this regimen does not achieve the desired effect, the oral antidiabetic agents should be discontinued, and mealtime analogue rapid-acting insulin can be added to the largest meal. Most clients will eventually require 4 injections with the basal-bolus regimen.

Marie, age 50, has type 1 diabetes and checks her blood glucose level several times every day. Her blood glucose level ranges from 250 to 280 mg/dL in the morning and is usually about 140 at lunch, about 120 at dinner, and about 100 at bedtime. In the morning, she takes 30 units of neutral protamine Hagedorn (NPH) insulin and 4 units of regular insulin, and before dinner she takes 18 units of NPH insulin and 4 units of regular insulin. Although she has had her insulin dose adjusted several times in the past month, it has had no effect on her high morning blood glucose level. What is your next course of action? A. Increase the evening NPH insulin dose by 2 more units B. Have her check her blood glucose level between 2 am and 4 am for the next several days C. Increase the morning regular insulin dose by 2 units D. Order a fasting blood sugar test

B. Have her check her blood glucose level between 2 am and 4 am Marie is experiencing the Somogyi phenomenon (rebound hyperglycemia). If her blood glucose level from 2 am to 4 am is greater than 70 mg/dL, the evening dose of NPH insulin should be increased and changed from before dinner to before bedtime. This should prevent most cases of nocturnal hypoglycemia, which results in morning hyperglycemia. Many providers prefer the longer acting insulins, such as insulin glargine (Lantus) and insulin detemir (Levemir), because they are mostly "peakless" and have less risk of hypoglycemia than NPH.

Sandra, age 28, has secondary obesity. Which of the following may have caused this? A. Taking in more calories than are expended B. Polycystic ovary syndrome C. Antihypertensive medications D. A sedentary lifestyle

B. Polycystic ovary syndrome Secondary obesity is rare; possible causes include Cushing disease, polycystic ovary syndrome, hypothalamic disease, hypothyroidism, and insulinoma. Some medications associated with weight gain include glucocorticoids, tricyclic antidepressants, and phenothiazines.

When teaching Marcy how to use her new insulin pump, you tell her that she needs to monitor her blood glucose level: A. At least once a day B. Only occasionally because glycemic levels are maintained very steadily C. At least 4 times a day D. On an as needed basis when she feels she needs to give herself an extra dose of insulin

C. At least 4 times a day. Clients using an insulin pump need to monitor their blood glucose levels at least 4 times a day. Clients can develop diabetic ketoacidosis in as little as 4 hours if there is mechanical failure of the pump because the only insulin used in the pump is rapid-acting.

Which class of antihypertensive agents may be problematic for clients with diabetes? A. ACE inhibitors B. Calcium channel blockers C. Beta blockers D. Alpha blockers

C. Beta blockers Beta blockers may be problematic in clients with diabetes because they block what is often the first sign of hypoglycemia—tachycardia. Many clients with diabetes have compelling indications (such as coronary artery disease) for the use of beta blockers. In these clients, the need for a beta blocker outweighs any risk that might occur. Decreasing the possibility of low blood sugar by selecting appropriate agents and adjusting dosages may be necessary. If a client with diabetes is on a beta blocker, it is important to explain that instead of tachycardia, he or she will notice other signs of hypoglycemia (such as sweating) that are not affected by beta blockers.

Which of the following is not a risk factor for diabetes mellitus type 2? A. BMI greater than 25 B. History of gestational diabetes. C. Caucasian race D. History of polycystic ovary syndrome (PCOS)

C. Caucasian race Caucasians are less likely than African Americans, Asian Americans, Latin Americans, and Native Americans to develop diabetes mellitus type 2.

The American Diabetes Association (ADA) recommends which of the following quarterly blood tests be performed on all clients with diabetes? A. TSH B. Liver function studies C. Glycated hemoglobin D. Serum glucose

C. Glycated hemoglobin The ADA recommends that the glycated hemoglobin (HbA1c) test be performed quarterly because it reports the serum glucose concentration of the previous 3 months. HbA1c can now be used for diagnosis of diabetes (greater than 6.5%). The ADA also recommends an annual urine test to assess for urine protein, which might be an early sign of kidney damage.

Which of the following statements about metformin is untrue? A. Metformin works by decreasing hepatic glucose production and increasing peripheral cell sensitivity to insulin. B. Metformin can cause lactic acidosis. C. Metformin is typically used in conjunction with other diabetes medications. D. Meformin cannot be used in patients with significant renal impairment.

C. Metformin is typically used in conjunction with other diabetes medications. Metformin is first-line treatment for diabetes type 2 and can be used as monotherapy.

Which is the only curative treatment option for primary hyperparathyroidism (PHPT)? A. Type II calcimimetic cinacalcet B. Hormone therapy C. Parathyroidectomy D. Bisphosphonates

C. Parathyroidectomy. Option 1: The type II calcimimetic cinacalcet treats the underlying cause of PHPT by binding to the calcium-sensing receptor on the surface of the parathyroid glands, which increases the sensitivity to extracellular calcium, which then reduces the excess secretion of parathyroid hormone (PTH). It is used for the treatment of secondary hyperparathyroidism but not for PHPT. Option 2: Hormone therapy is not used by itself. Low doses of estrogen have been shown to reduce calcium, prevent bone loss, and improve bone density. Option 3: The only curative treatment option for PHPT is a parathyroidectomy. It is successful in 90% to 98% of cases. Option 4: Although the first generation of bisphosphonates was found to be ineffective for treatment of the skeletal manifestations of PHPT, the newer bisphosphonates, such as alendronate (Fosamax), increase bone density a little, but they do not affect parathyroid hormone (PTH) secretion and thus will not reduce serum calcium.

Jenny, age 46, has hypertension that has been controlled with hydrochlorothiazide 50 mg every day for the past 3 years. She is 5 ft 8 in tall and weighs 220 lb. Her fasting blood sugar (FBS) is 300 mg/dL, serum cholesterol level is 250 mg/dL, serum potassium level is 3.4 mEq, and she has 4+ glucosuria. Your next course of action would be to: A. Discontinue her hydrochlorothiazide B. Order a glucose tolerance test (GTT) C. Repeat her FBS and do a glycated hemoglobin (HbA1c) D. Start insulin therapy

C. Repeat her FBS and do a glycated hemoglobin (HbA1c) Option 1: Hyperglycemia can be an adverse reaction to high doses of hydrochlorothiazide, but the first action would be to repeat the FBS. If it is still high on a second reading, the diuretic should be reduced. Option 2: A GTT to confirm a diagnosis of diabetes is usually not needed. Diabetes is not usually diagnosed with a single high glucose reading unless symptoms of polyphagia, polydipsia, and polyuria are present. Option 3: Jenny's FBS should be repeated along with an HbA1c. An HbA1c of greater than 6.5% can now be used to diagnosis diabetes. Option 4: Insulin therapy would not be started until Jenny was given a positive diagnosis, and even then oral antidiabetic agents would be considered first.

Jeffrey, age 17, has gynecomastia. You should also assess him for: a. obesity b. endocrine abnormalities c. testicular cancer d. TB

C. Testicular cancer Gynecomastia may be the first sign of testicular cancer. It is also associated with breast, adrenal, pituitary, lung, and hepatic malignancies. Hypogonadism produces low testosterone levels in men with normal estrogen levels. Alteration in breast tissue responsiveness to hormonal activity can result in gynecomastia. Gynecomastia can occur secondary to cirrhosis, chronic obstructive lung disease, malnutrition, hyperthyroidism and other endocrine imbalances, tuberculosis, and chronic renal disease.

Mason, age 52, has DM and is overweight. You now find that he is hypertensive. How should you treat his hypertension? A. You should treat it the same as in a client without diabetes. B. Because insulin affects most antihypertensive drugs, you should try diet and exercise first before ordering any antihypertensives. C. You should treat it very aggressively, preferably with angiotensin-converting enzyme (ACE) inhibitors D. You should initiate therapy when the blood pressure is 5 to 10 mm Hg more than the conventional therapeutic guidelines.

C. You should treat it very aggressively, preferably with angiotensin-converting enzyme (ACE) inhibitors Option 1: Because hypertension is implicated in accelerating the microangiopathy of diabetes (especially retinopathy and nephropathy), according to the Eighth Joint National Committee (JNC 8) therapy should be initiated when the patient has a blood pressure at or above 140/90 mm Hg. Other organizations recommend the lower goal of 130/80 in clients with DM. Option 2: Owing to the beneficial effects of reducing albuminuria and glomerular pressure, angiotensin-converting enzyme (ACE) inhibitors are the drugs of choice in clients with DM with hypertension. Option 3: ACE inhibitors should be initiated in patients with diabetes and an elevated blood pressure and are recommended in normotensive patients with albuminuria. Option 4: Depending on the guidelines, therapy should be initiated when blood pressure reaches levels equal to or 10 mm Hg less than conventional therapeutic guidelines.

Jennifer has diabetes mellitus (DM) and is injecting 30 units of Novolin 70/30 with breakfast and 8 units at bedtime. She is complaining that she woke up in the middle of the night with palpitations and swearing. Based on this information, what do you recommend? A. Decreasing the am dose of 70/30 B. Decreasing the pm dose of 70/30 C. Eating a snack before going to bed D. Changing the time of the nighttime insulin injection

D. Changing the time of the nighttime insulin injection The Novolin 70/30 should be given before a meal. Jennifer was not given appropriate instructions on the timing of the injections. She should inject her nighttime dose about 30 minutes prior to the evening meal. Injecting this type of insulin before bed will cause the blood sugar to drop because 30% of the insulin is regular insulin.

A patient presents to your primary care office complaining of polydipsia, polyuria, and polyphagia. Which of the following diagnoses would not be in your differential diagnosis? A. Diabetes mellitus (DM) B. Diabetes insipidus (DI) C. Psychiatric disorders D. Hyperthyroidism

D. Hyperthyroidism Symptoms of hyperthyroidism could include polyphagia due to increased metabolism, but symptoms generally are weight loss, tachycardia, and other physical manifestations of increased metabolism.

An elderly client presents with atrial fibrillation. Which of the following lab tests is important in forming the diagnosis? A. CBC B. C-reactive protein (CRP) C. Comprehensive metabolic panel (CMP) D. TSH

D. TSH Option 1: A CBC is not specific. Option 2: A CRP will detect inflammation and the possibility of heart disease but not atrial fibrillation. Option 3: A CMP is not specific. Option 4: Atrial fibrillation is a common presentation in elderly clients with hyperthyroidism. If the TSH is suppressed, a free thyroxine (T4) and triiodothyronine (T3) should be drawn.

Jay has had diabetes for 10 years. He recently had a physical and was told he has some evidence of renal nephropathy. What is the first manifestation of this renal dysfunction? a. Microalbuminuria b. Development of Kimmelstiel-Wilson nodules c. Decreased serum urea nitrogen levels. d. Decreased serum creatinine levels.

a. microalbuminuria Microalbuminuria is the first symptom indicative of nephropathy in clients who have had diabetes for about 10 years (although some studies suggest 5 years). There is increased permeability of the capillaries, with resultant leakage of albumin into the glomerular filtrate, causing microalbuminuria.

Jason, age 14, appears with tender discoid breast tissue enlargement (2-3 cm) beneath the areola. Your next action would be to: a. perform watchful waiting for 1 year b. order an US c. obtain lab tests d. refer Jason to an endocrinologist.

a. perform watchful waiting for 1 year Pubertal gynecomastia is common and is characterized by tender discoid breast tissue enlargement of about 2 to 3 cm in diameter beneath the areolae. The swelling usually subsides spontaneously within a year, and watchful waiting along with reassurance is recommended for that time period.

Mindy is scheduled to have an oral glucose tolerance test (OGTT). For 3 days before the test, she is instructed to dc many of her medications. Which one is it safe to continue? a. vitamin C b. ASA c. Calcium d. her oral contraceptives

c. Calcium Calcium does not affect an OGTT. The following medications may interfere with the results of an OGTT and should be discontinued for 3 days before the test: vitamin C, aspirin, oral contraceptives, corticosteroids, synthetic estrogens, phenytoin (Dilantin), thiazide diuretics, and nicotinic acid.


Kaugnay na mga set ng pag-aaral

Business Law - Practice SET - part 2

View Set

CH 12: Factorial Experimental Designs

View Set

Meggs History of Graphic Design ch.9

View Set

3e - Math, chapitre 2 : Pythagore

View Set

English III: Coming of Age in the Dawnland from 1491

View Set